143 FINAL - Mod 4: Immune Disorders (PRACTICE QUESTIONS)

Lakukan tugas rumah & ujian kamu dengan baik sekarang menggunakan Quizwiz!

A client is receiving a transfusion of packed red blood cells. Shortly after initiation of the transfusion, the client begins to exhibit signs and symptoms of a transfusion reaction. The client is suffering from which type of hypersensitivity? A. Anaphylactic (type 1) B. Cytotoxic (type II) C. Immune complex (type III) D. Delayed type (type IV)

ANS: B Rationale: A type II hypersensitivity reaction resulting in red blood cell destruction is associated with blood transfusions. This type of reaction does not result from types I, III, or IV reactions.

Which type of immunity becomes active as a result of infection by a specific microorganism? A. Artificially acquired active immunity B. Naturally acquired active immunity C. Naturally acquired passive immunity D. Artificially acquired passive immunity

B. Naturally acquired active immunity occurs as a result of an infection by a specific microorganism. Artificially acquired active immunity results from the administration of a killed or weakened microorganism or toxoid. Passive immunity develops when ready-made antibodies are given to a susceptible individual.

The nurse is teaching the client with HIV about therapy. Which elements are essential for the nurse to include in the teaching plan? Select all that apply. A. Clients rarely respond to medication therapy. B. The goal of antiretroviral therapy is to prevent opportunistic infections. C. Medication therapy is rarely effective. D. The CD4 count is the major indicator of immune function and guides therapy. E. Antiretroviral therapy targets different stages of the HIV life cycle.

D, E. The CD4 count is the major indicator of immune function. Antiretroviral therapy in HIV targets different stages of the HIV life cycle. Therapy does not prevent opportunistic infections. Medication therapy is effective, and most clients respond well to it.

The teaching plan for the client with rheumatoid arthritis includes rest promotion. Which of the following would the nurse expect to instruct the client to avoid during rest periods? 1. Proper body alignment. 2. Elevating the part. 3. Prone lying positions. 4. Positions of flexion.

4. Positions of flexion should be avoided to prevent loss of functional ability of affected joints. Proper body alignment during rest periods is encouraged to maintain correct muscle and joint placement. Lying in the prone position is encouraged to avoid further curvature of the spine and internal rotation of the shoulders.

During a mumps outbreak at a local school, a teacher has been exposed. The client has previously been immunized for mumps, and consequently possesses: A. acquired immunity. B. natural immunity. C. phagocytic immunity. D. humoral immunity.

ANS: A Rationale: Acquired immunity usually develops as a result of prior exposure to an antigen, often through immunization. When the body is attacked by bacteria, viruses, or other pathogens, it has three means of defense. The first line of defense, the phagocytic immune response, involves the WBCs that have the ability to ingest foreign particles. A second protective response is the humoral immune response, which begins when the B lymphocytes transform themselves into plasma cells that manufacture antibodies. The natural immune response system is rapid, nonspecific immunity present at birth.

After the completion of testing, a 7-year-old client's allergies have been attributed to the family's cat. When introducing the family to the principles of avoidance therapy, the nurse should promote which action? A. Removing the cat from the family's home B. Administering over-the-counter antihistamines to the client regularly C. Keeping the cat restricted from the client's bedroom D. Maximizing airflow in the house

ANS: A Rationale: In avoidance therapy, every attempt is made to remove the allergens that act as precipitating factors. Fully removing the cat from the environment is preferable to just keeping the cat out of the client's bedroom. Avoidance therapy does not involve improving airflow or using antihistamines.

A client has been brought to the emergency department after being found unresponsive, and anaphylaxis is suspected. The care team should attempt to assess for which potential causes of anaphylaxis? Select all that apply. A. Foods B. Medications C. Insect stings D. Autoimmunity E. Environmental pollutants

ANS: A, B, C Rationale: Substances that most commonly cause anaphylaxis include foods, medications, insect stings, and latex. Pollutants do not commonly cause anaphylaxis and autoimmune processes are more closely associated with types II and III hypersensitivities.

A client is in the primary infection stage of human immunodeficiency virus (HIV). Which statement regarding this client's current health status is most accurate? A. The client's HIV antibodies are successfully, but temporarily, killing the virus. B. The client is infected with HIV but lacks HIV-specific antibodies. C. The client's risk for opportunistic infections is at its peak. D. The client may or may not develop long-standing HIV infection.

ANS: B Rationale: The period from infection with HIV to the development of HIV-specific antibodies is known as primary infection. The virus is not being eradicated and infection is certain. Opportunistic infections emerge much later in the course of the disease.

A community health nurse is performing a visit to the home of a client who has a history of rheumatoid arthritis (RA). On which aspect of the client's health should the nurse focus most closely during the visit? A. Understanding of rheumatoid arthritis B. Risk for cardiopulmonary complications C. Social support system D. Functional status

ANS: D Rationale: The client's functional status is a central focus of home assessment of the client with RA. The nurse may also address the client's understanding of the disease, complications, and social support, but the client's level of function and quality of life are a primary concern.

Teaching that the nurse will plan for the patient with SLE includes: A. Ways to avoid exposure to sunlight B. Increasing dietary protein and carbohydrate intake C. The necessity of genetic counseling before planning a family D. The use of non pharmacologic pain interventions instead of analgesics

Answer: A Ways to avoid exposure to sunlight

The nurse completes a history and physical assessment on a client with acquired immune deficiency syndrome (AIDS) who was admitted to the hospital with respiratory complications. The nurse knows to assess for what common infection (80% occurrence) in persons with AIDS? A. Mycobacterium tuberculosis B. Pneumocystis pneumonia C. Cytomegalovirus D. Legionnaires disease

B. Pneumocystis pneumonia (PCP) is one of the first and most common opportunistic infections associated with AIDS. It may be present despite the absence of crackles. If untreated, PCP progresses to cause significant pulmonary impairment and respiratory failure.

A nurse is taking the health history of a newly admitted client. Which of the following conditions would NOT place the client at risk for impaired immune function? A. History of radiation therapy B. Surgical removal of the appendix C. Surgical history of a splenectomy D. Previous organ transplantation

B. Removal of the appendix would have no direct effect on the immune system. Organ transplantation requires immunosuppressive drugs, which cause impaired immune function. Radiation therapy destroys lymphocytes. The spleen is an important part of the immune system, and removal of it increases the client's risk for poor immune function.

In teaching a patient with SLE about the disorder, the nurse knows that the pathophysiology of SLE includes: A. Circulating immune complexes formed from IgG autoantibodies reacting with IgG B. An autoimmune T-cell reaction that results in destruction of the deep dermal skin layer C. Immunologic dysfunction leading to chronic inflammation in the cartilage and muscles D. The production of a variety of autoantibodies directed against components of the cell nucleus

Answer: D Rationale: Systemic lupus erythematosus (SLE) is characterized by the production of many autoantibodies against nucleic acids (e.g., single-and double-stranded DNA), erythrocytes, coagulation proteins, lymphocytes, platelets, and many other self-proteins. Autoimmune reactions characteristically are directed against constituents of the cell nucleus (e.g., antinuclear antibodies [ANAs]), particularly DNA. Circulating immune complexes containing antibody against DNA are deposited in the basement membranes of capillaries in the kidneys, heart, skin, brain, and joints. Complement is activated, and inflammation occurs. The overaggressive antibody response is also related to activation of B and T cells. The specific manifestations of SLE depend on which cell types or organs are involved. SLE is a type III hypersensitivity response.

A client has been living with seasonal allergies for many years, but does not take antihistamines, stating, When I was young, I used to take antihistamines, but they always put me to sleep. How should the nurse best respond? A. Newer antihistamines are combined with a stimulant that offsets drowsiness. B. Most people find that they develop a tolerance to sedation after a few months. C. The newer antihistamines are different than in years past, and cause less sedation. D. Have you considered taking them at bedtime instead of in the morning?

ANS: C Rationale: Unlike first-generation H1 receptor antagonists, newer antihistamines bind to peripheral rather than central nervous system H1 receptors, causing less sedation, if any at all. Tolerance to sedation did not usually occur with first-generation drugs, and newer antihistamines are not combined with a stimulant. Although taking an antihistamine at bedtime may be a suitable option for the client, it is not the nurse's best response because it does not inform the client of the newer antihistamines, which cause little or no sedation and thus could be taken any time of day.

The nurse is assessing a patient for altered immunocompetence. Which findings would indicate that the patient is at risk for developing an immunocompetence-associated illness? Select all that apply. 1. Slow wound healing and easy bruising 2. Bursitis and muscle cramps 3. Heart palpitations 4. Heartburn and increased flatus 5. Mouth sores and oral patches

Answer: 1, 5 Explanation: 1. Assessment data that could indicate an immunocompetence-associated illness includes slow wound healing and easy bruising. 2. Bursitis and muscle cramps have little association with altered immunocompetence. 3. Heart palpitations have little association with altered immunocompetence. 4. Heartburn and increased flatus are not associated with altered immunocompetence. 5. Mouth sores and oral patches are related to immunocompetence.

The registered nurse (RN) will educate the client to implement what action as soon as the first dose of the epipen is given? A) Take a diphenhydramine tablet B) Call for emergency care immediately C) Use an albuterol inhaler D) Take a steroid by mouth

B. Epinephrine is a sympathomimetic drug. This drug stimulates alpha and beta receptors, resulting in vasoconstriction. For anaphylaxis, the drug is administered by an epinephrine auto-injector, also known as an 'epi-pen'. Epinephrine is the priority drug of treatment for an anaphylactic event. This drug is administered intramuscularly and has a rapid onset, but a 10 to 20 minute duration. For the emergency administration of this drug, skin prep is not indicated. The drug is administered into the outer muscle of the thigh at a 90 degree angle. Once inserted into the muscle, the syringe is held in place for 10 seconds allowing the drug/syringe to self-inject. The drug is repeated every 5 to 15 minutes until all anaphylactic symptoms have resolved. Emergency care should be sought immediately after the administration of the epi-pen as the drug has a short duration. This drug is stored at room temperature, in a dark location. Expected side-effects of this drug include: dizziness, palpitations, and tachycardia. Once the symptoms are resolved the next drugs to be administered (in order) include diphenhydramine, albuterol inhaler, and steroids.

A client is brought to the emergency department (ED) in a state of anaphylaxis. What is the ED nurse's priority for care? A. Provide psychosocial support. B. Administer medications as prescribed. C. Protect the client's airway. D. Monitor the client's level of consciousness (LOC).

C. Anaphylaxis severely threatens a client's airway; the nurse's priority is preserving airway patency and breathing pattern. This is a higher priority than other valid aspects of care, including medication administration, psychosocial support, and assessment of LOC.

A client develops and anaphylactic reaction after receiving morphine. The nurse would take which actions? Select all that apply. A) Administer oxygen B) Quickly assess the client's respiratory status C) Document the event, interventions, and client's response D) Leave the client briefly to contact a primary health care provider E) Keep the client supine regardless of the blood pressure readings F) Start an IV infusion of D5W and administer a 500-mL bolus

A, B, C.

Which general instruction will the registered nurse (RN) provide the client before dispensing a prescribed epipen? Select all that apply. A) Keep this drug with you at all times in the event it is needed B) It is important to keep the epi-pen in a dark location C) It is less painful if the drug is administered chilled, so keep it in the refrigerator D) You can expect to feel dizzy and a fast heartbeat after giving the drug E) You will need to avoid eating or drinking for 24 hours after receiving this drug

A, B, D. Epinephrine is a sympathomimetic drug. This drug stimulates alpha and beta receptors, resulting in vasoconstriction. For anaphylaxis, the drug is administered by an epinephrine auto-injector, also known as an 'epi-pen'. Epinephrine is the priority drug of treatment for an anaphylactic event. This drug is administered intramuscularly and has a rapid onset, but a 10 to 20 minute duration. For the emergency administration of this drug, skin prep is not indicated. The drug is administered into the outer muscle of the thigh at a 90 degree angle. Once inserted into the muscle, the syringe is held in place for 10 seconds allowing the drug/syringe to self-inject. The drug is repeated every 5 to 15 minutes until all anaphylactic symptoms have resolved. Emergency care should be sought immediately after the administration of the epi-pen as the drug has a short duration. This drug is stored at room temperature, in a dark location. Expected side-effects of this drug include: dizziness, palpitations, and tachycardia.

Which education point will the registered nurse (RN) provide the client prescribed an immunosuppressant? Select all that apply. A) Use a soft bristled toothbrush and be gentle when brushing your teeth. B) Notify the healthcare provider if you maintain a fever of 100.3F for more than 2 days C) Notify the HCP if you notice blood in stool or from gums D) As long as the fresh flowers have been dipped in sugar water, you can keep them in your home. E) It is important to remain current with all vaccinations while on this drug.

A, C. Immunosuppressants are a class of drugs that work to suppress or reduce the function of the body's immune system. This drug class is used to treat autoimmune diseases where the body is attacking itself. The drug class is also used to prevent the rejection of an organ transplant. The actions of this drug class result in a decrease in white blood cells and platelets. This reduction increases the risk of infection and bleeding. Education for the individual includes reporting immediately any indication of an infection, including a fever greater than 100.3 F (38 C). The individual is taught to avoid crowds, as well as fresh flowers or fruit. Live vaccines and pregnancies are contraindicated during this drug therapy. The individual is educated to use a soft bristled toothbrush and to report any signs of bleeding (bleeding gums, petechiae, purpura, melena, and/or hematemesis).

What education should the nurse provide to the patient taking long-term corticosteroids? A. The patient should not stop taking the medication abruptly and should be weaned off of the medication. B. Corticosteroids are relatively safe drugs with very few side effects. C. The patient should discontinue using the drug immediately if weight gain is observed. D. The patient should take the medication only as needed and not take it unnecessarily.

A. Patients who receive high-dose or long-term corticosteroid therapy must be cautioned not to stop taking the medication suddenly. Doses are tapered when discontinuing this medication to avoid adrenal insufficiency.

A client with HIV will be started on a medication regimen of three medications. What class of drugs will the nurse instruct the client about? A. Reverse transcriptase inhibitors B. Hydroxyurea C. Anticholinergics D. Disinhibitors

A. Reverse transcriptase inhibitors are drugs that interfere with the virus' ability to make a genetic blueprint. A protease inhibitor is a drug that inhibits the ability of virus particles to leave the host cell. The integrase inhibitors are a class of drug that prevents the incorporation of viral DNA into the host cell's DNA. Hydroxyurea is a drug that is used as an adjunct therapy that tries to halt the progression of AIDS.

A client recently diagnosed with an inflammatory bowel disease is given sulfasalazine. Which of the following side effects of the drug should the nurse educate the client? A) A yellow or orange color of the skin and urine are common side effects B) Chest pain C) Constipation is an expected side effect D) Urinary retention is an expected side effect

A. Sulfasalazine is a common treatment for inflammatory bowel disease (IBD). The common side effects of sulfasalazine are orange or yellowing color of the skin and urine. The patient must be educated that the health care provider (HCP) will be monitoring and do not discontinue the medication. Since sulfasalazine is a sulfa-drug, patients with IBD must be asked of any allergies. Sulfasalazine does not cure the disease, but does improve symptoms of the disease. Other medications for IBD may include dicyclomine, which acts as an anti-diarrheal agent.

In regards to HIV education, which is the most important objective for the nurse? A) Prevention B) Interventions against opportunistic infection C) Treatment regimen compliance D) Determine at-risk behaviors

A. The primary goal is preventing the spread of the virus since there is no cure and it can be transmitted at all stages. The ways to prevent transmission of HIV are straightforward. Utilizing safe sex practices with use of condoms or through abstinence is the most effective way to prevent the spread. Ensuring universal precautions are in effect while working as a healthcare worker is another way to prevent contracting HIV. Another means of prevention of HIV is staying sober-minded not to participate in risky drug use behavior. The Centers for Disease Control (CDC) mentions HIV testing and preexposure prophylaxis as well.

After teaching a client how to self-administer epinephrine, the nurse determines that the teaching plan has been successful when the client demonstrates which action? A. Jabs the autoinjector into the outer thigh at a 90-degree angle B. Maintains pressure on the auto-injector for about 30 seconds after insertion C. Avoids massaging the injection site after administration D. Pushes down on the grey release cap to administer the medication

A. To self-administer epinephrine, the client should remove the autoinjector from its carrying tube, grasp the unit with the black tip (injecting end) pointed downward, form a fist around the device, and remove the gray safety release cap. Then the client should hold the black tip near the outer thigh and swing and jab firmly into the outer thigh at a 90-degree angle until a click is heard. Next, the client should hold the device firmly in place for about 10 seconds, remove the device, and massage the site for about 10 seconds.

The nurse is providing care for a client who has experienced a type I hypersensitivity reaction. Which client would have this type of reaction? A. A client with an anaphylactic reaction after a bee sting B. A client with a skin reaction resulting from adhesive tape C. A client with a diagnosis of myasthenia gravis D. A client with rheumatoid arthritis

ANS: A Rationale: Anaphylactic (type I) hypersensitivity is an immediate reaction mediated by immunoglobulin E antibodies and requires previous exposure to the specific antigen. Skin reactions are more commonly type IV, and myasthenia gravis is thought to be a type II reaction. Rheumatoid arthritis is not a type I hypersensitivity reaction.

A client with a history of dermatitis takes corticosteroids on a regular basis. The nurse should assess the client for which complication of therapy? A. Immunosuppression B. Agranulocytosis C. Anemia D. Thrombocytopenia

ANS: A Rationale: Corticosteroids, such as prednisone, can cause immunosuppression. Corticosteroids do not typically cause agranulocytosis, anemia, or low platelet counts. Agranulocytosis, which is a decrease in granulocytes, a type of white blood cell, may be caused by antibiotics, antithyroid drugs, or nonsteroidal anti-inflammatory drugs. Anemia, which is a decrease in red blood cells, may be caused by antibiotics or nonsteroidal anti-inflammatory drugs. Thrombocytopenia, which is a decrease in platelets, may be caused by antibiotics.

A client has been admitted to the emergency department with signs of anaphylaxis following a bee sting. The nurse knows that if this is a true allergic reaction the client will present with what alteration in laboratory values? A. Increased eosinophils B. Increased neutrophils C. Increased serum albumin D. Decreased blood glucose

ANS: A Rationale: Higher percentages of eosinophils are considered moderate to severe eosinophilia. Moderate eosinophilia is defined as 15% to 40% eosinophils and is found in clients with allergic disorders. Hypersensitivity does not result in hypoglycemia or increased albumin and neutrophil counts.

An office worker eats a cookie that contains peanut butter. The worker begins wheezing, with an inspiratory stridor and air hunger, and the occupational health nurse is called to the office. The nurse should recognize that the worker is likely suffering from which type of hypersensitivity? A. Anaphylactic (type 1) B. Cytotoxic (type II) C. Immune complex (type III) D. Delayed-type (type IV)

ANS: A Rationale: The most severe form of a hypersensitivity reaction is anaphylaxis. An unanticipated severe allergic reaction that is often explosive in onset, anaphylaxis is characterized by edema in many tissues, including the larynx, and is often accompanied by hypotension, bronchospasm, and cardiovascular collapse in severe cases. Type II, or cytotoxic, hypersensitivity occurs when the system mistakenly identifies a normal constituent of the body as foreign. Immune complex (type III) hypersensitivity involves immune complexes formed when antigens bind to antibodies. Type III is associated with systemic lupus erythematosus, rheumatoid arthritis, certain types of nephritis, and bacterial endocarditis. Delayed-type (type IV), also known as cellular hypersensitivity, occurs 24 to 72 hours after exposure to an allergen.

A nurse has given an 8-year-old client the scheduled vaccination for rubella. This vaccination will cause the client to develop which expected and desired condition? A. Natural immunity B. Passive acquired immunity C. Cellular immunity D. Mild hypersensitivity

ANS: B Rationale: Passive/adaptive acquired immunity usually develops as a result of vaccination or contracting a disease. Natural immunity is present at birth and provides a nonspecific response to any foreign invader. Immunizations do not activate the process of cellular immunity. Cellular immunity is part of the innate/natural immunity response, which involves T cells that neutralize components of the threat within the cell itself. Hypersensitivity is infrequent, and adverse reactions (i.e., urticaria, anaphylaxis) to vaccine administration are rare.

A nurse is working with a client who was diagnosed with HIV several months earlier. This client will be considered to have AIDS when the CD4+ T-lymphocyte cell count drops below what threshold? A. 75 cells/mm3 of blood B. 200 cells/mm3 of blood C. 325 cells/mm3 of blood D. 450 cells/mm3 of blood

ANS: B Rationale: When CD4+ T-cell levels drop below 200 cells/mm3 of blood, the person is said to have AIDS.

A nurse is caring for a client hospitalized with AIDS. A friend comes to visit the client and privately asks the nurse about the risk of contracting HIV when visiting the client. What is the nurse's best response? A. Do you think that you might already have HIV? B. Your immune system is likely very healthy. C. AIDS isn't transmitted by casual contact. D. You can't normally contract AIDS in a hospital setting.

ANS: C Rationale: AIDS is commonly transmitted by contact with blood and body fluids. Clients, family, and friends must be reassured that HIV is not spread through casual contact. A healthy immune system is not necessarily a protection against HIV. A hospital setting does not necessarily preclude HIV infection.

A nurse has included the nursing diagnosis of Risk for Latex Allergy Response in a client's plan of care. The presence of which chronic health problem would most likely prompt this diagnosis? A. Herpes simplex B. Human immunodeficiency virus (HIV) C. Spina bifida D. Hypogammaglobulinemia

ANS: C Rationale: Clients with spina bifida are at a particularly high risk for developing a latex allergy. Clients with spina bifida are at high risk because they have had multiple surgeries, multiple urinary catheterization procedures, and other treatments involving use of latex products, and latex allergy develops as a result of repeated exposure to the proteins and polypeptides in natural rubber latex. Clients with herpes simplex, HIV, or hypogammaglobulinemia (decreased level of gamma immunoglobulins) are less likely than clients with spina bifida to have as many surgeries or other treatments that would expose them to latex.

A client diagnosed with rheumatoid arthritis is prescribed methotrexate. Which of these interventions are priority to include in the discharge teaching? Select all that apply. A) Report a temp of 100.3 F to other HCP B) Avoid crowds and large gatherings of people C) Do not take any live vaccines D) Perform high impact exercises regularly E) Watch for signs of bleeding

A, B, C, E. Methotrexate is an immunosuppressant that stops folic acid metabolism which stops cellular reproduction in the fastest replicating cells. Methotrexate suppresses B and T lymphocytes, therefore white blood cells (WBCs). The client has a high risk for infection, a low platelet count, and fetal death. The client should report a temperature of 100.3º F or 38º C to the health care provider (HCP), avoid crowds and large gatherings of people, do not take any live vaccines, only inactivated vaccines, and watch for signs of bleeding such as melena, purpura, petechiae, hematemesis, and bleeding gums. Clients are encouraged to utilize birth control until 3 months after treatment is completed because of the high risk of fetal demise. Clients should avoid using razors and hard toothbrushes to prevent bleeding.

Which of the following pharmacological agents decreases prostaglandin response to reduce pain and inflammation? A) Ibuprofen B) Acetaminophen C) Morphine D) Fentanyl

A. Ibuprofen is a non-steroidal anti-inflammatory drug (NSAID) which decreases prostaglandin response to reduce pain and inflammation. NSAIDs also decrease platelet aggregation, especially aspirin, which leads to a major bleeding risk. Educate the client to never take two NSAIDs simultaneously to decrease the bleeding risk, and use the lowest dose for the shortest time possible. NSAIDs must also be avoided in clients with acid reflux and peptic ulcer disease unless the client is on acid-reducing drugs. Since NSAIDs block prostaglandins, which protect the lining of the stomach, avoid using NSAIDs because NSAIDs increase gastrointestinal bleeding.

What type of immunoglobulin does the nurse recognize that promotes the release of vasoactive chemicals such as histamine when a client is having an allergic reaction? A. IgE B. IgM C. IgA D. IgG

A. IgE promotes the release of vasoactive chemicals such as histamine and bradykinin in allergic, hypersensitivity, and inflammatory reactions. IgG neutralizes bacterial toxins and accelerates phagocytosis. IgA interferes with the entry of pathogens through exposed structures or pathways. IgM agglutinates antigens and lyses cell walls.

What test will the nurse assess to determine the client's response to antiretroviral therapy? A. Viral load B. Complete blood count C. Enzyme immunoassay D. Western blotting

A. Viral load should be measured at baseline and on a regular basis thereafter because viral load is the most important indicator of response to ART. The other tests are not used in this way.

A 5-year-old client has been diagnosed with a severe food allergy. Which instruction should the nurse include when educating the parents about this client's allergy and care? A. Wear a medical identification bracelet. B. Know how to use the antihistamine pen. C. Know how to give injections of lidocaine. D. Avoid live attenuated vaccinations.

ANS: A Rationale: The nurse also advises the parents to have the client wear a medical identification bracelet and to be able to identify symptoms of food allergy. Clients and their families do not carry antihistamine pens, they carry epinephrine pens. Lidocaine is not self-administered to treat allergies. The client may safely be vaccinated.

A nurse is providing care for a client who has just been diagnosed with early-stage rheumatoid arthritis (RA). The nurse should anticipate the administration of which medication? A. Hydromorphone B. Methotrexate C. Allopurinol D. Prednisone

ANS: B Rationale: Once the diagnosis of RA is made, treatment should begin with either a nonbiologic or biologic disease-modifying antirheumatic drug (DMARD). Recommended treatment guidelines include beginning with the nonbiologic DMARDs (methotrexate, leflunomide, sulfasalazine) or hydroxychloroquine within 3 months of disease onset. Allopurinol is used to treat gout. Opioids are not indicated in early RA. Prednisone is used in unremitting RA.

What statement made by the client prescribed hydroxychloroquine for the treatment of lupus will alert the registered nurse (RN) to the need to reinforce education about this drug? A) I can expect an improvement in my energy level while taking this drug. B) This drug should result in less inflammation in my body over time. C) This drug should cause a huge improvement in my health by the end of 2 weeks. D) It is important that I have eye appointments every 6-12 months while on this drug.

C. Hydroxychloroquine is a quinolone drug with immunosuppressant actions used in the treatment of lupus. The immunosuppressant action of this drug assists in decreasing the fatigue and inflammation that occurs with this disease. The achievement of therapeutic level is slow for this drug, taking several months to achieve. The drug can be taken with food to decrease the effects of nausea, and should be taken whole (do not crush, chew, or split). The major side-effect of this drug is vision problems and retinal damage. It is important that the individual know to report any vision changes immediately to the healthcare provider. Because of the risk of retinal damage, it is recommended that the individual receive an eye examination every 6 to 12 months while receiving this drug.

What type of hypersensitivity reaction is exhibited by a client that has been diagnosed with rheumatoid arthritis? A) Type I reaction B) Type II reaction C) Type III reaction D) Type IV reaction

C. Hypersensitivity reactions are immune responses that are exaggerated or inappropriate against an antigen or allergen. In a type III reaction, excess antigens cause immune complexes to form in the blood. These can precipitate in various tissues, such as skin, kidneys, joints, and the small blood vessels. These immune complexes that are deposited trigger inflammation resulting in damage to the tissue or vessel. Examples of type III reaction include: rheumatoid arthritis, systemic lupus erythematosus, and glomerulonephritis.

A client is taking a corticosteroid for the treatment of systemic lupus erythematosus. When the nurse is providing instructions about the medication to the client, what priority information should be included? A. This medication is commonly used for many inflammatory reactions and is relatively safe. B. The client should be alert for joint aches. C. Be alert for signs and symptoms of infection and report them immediately to the physician. D. If the client experiences nausea, omit the dose.

C. Instruct the client about signs and symptoms of and the increased risk for infection. Instruct the client to report signs and symptoms of infection immediately to the physician. Early treatment promotes a shorter duration of illness and reduced complication. Tell the client to avoid high-risk activities, such as being in crowds, during periods of immunosuppression. The client should not omit a dose if nausea is experienced; he may take the medication with food. There are many side effects and required laboratory work to detect the side effects from immunosuppressive therapy. Joint aches are vague symptoms and are not a priority for reporting purposes.

When presenting a comparison of diphenhydramine and loratadine, what primary advantage will the registered nurse (RN) provide for choosing loratadine over diphenhydramine for the treatment of seasonal allergies? A) Loratadine costs loss than diphenhydramine B) Loratadine is short-acting than diphenhydramine C) Loratadine has fewer side effects than diphenhydramine D) Loratadine tastes better than diphenhydramine

C. Loratadine is an antihistamine drug that reduces histamine in the body. This drug has less side-effects compared to diphenhydramine. This makes this drug a good choice for the treatment of allergies. Loratadine does not prevent hives or other allergic skin reactions as diphenhydramine does. This drug has a 1 hour onset and a 24 hour duration. Common side-effects of this drug include: headache, dry mouth, and diarrhea.

A 21-year-old male is being started on zidovudine (AZT) a Nucleotide Reverse Transcriptase Inhibitor (NRTI) for treatment of HIV/AIDS. Which of the following statements made by the patient indicates that he has understood the patient teaching? A) AZT inactivates the virus and prevents recurrence of the disease. B) AZT therapy may result in the development of AZT-resistant strains. C) AZT slows the progression of the disease but does not cure it. D) AZT prevents the occurrence of opportunistic infections.

C. Zidovudine slows the progression of the disease but does not cure the disease. Zidovudine does not inactivate the virus. Zidovudine does not result in resistant strains.Zidovudine does not prevent the occurrence of opportunistic infections.

While reviewing the blood lab value results, which lab value is diagnostic for systemic lupus erythematosus (SLE)? Select all that apply. A) Elevated WBCs B) Decreased sodium C) Decreased creatinine D) Elevated ESR E) Elevated CRP

D, E. Systemic lupus erythematosus (SLE) should be suspected in any childbearing age woman that presents with the most common signs and symptoms. It may be difficult initially to diagnose SLE as the lab results mimic other connective tissue disorders. SLE attacks multiple organs with the kidneys being the first. This will cause an increased creatinine level greater than 1.3 mg/dL. Other lab elevations expected because of the autoimmune response is an elevated erythrocyte sedimentation rate (ESR) and C-reactive protein (CRP). As a direct attack on the blood cells, the white blood cell will present below the expected 5000 per mm3.

What education will the registered nurse (RN) provide the client receiving diphenhydramine to treat allergies? A) Take this medication once daily before breakfast with a full glass of water. B) Take this medication every 2 hours until the allergy symptoms are gone. C) Avoid eating any high carb foods while taking this medication. D) Avoid driving or operating machinery while taking this medication.

D. Diphenhydramine is an antihistamine drug that works to block histamine. Histamine causes increased mucus and secretions, as well as sneezing, runny nose, and itching. Diphenhydramine is used to treat anaphylaxis as well as minor allergies. The major side-effect of this drug is sedation and drowsiness. For this reason, the individual should not drive or operate machinery while taking diphenhydramine. This drug is contraindicated in individuals with a history of peptic ulcer disease, small bowel obstruction, urinary retention, and closed angle glaucoma.

What statement made by the student best explains type II reaction, as with a hemolytic blood transfusion? A) The body develops inflammation to excess antigens that cause immune complexes to form. B) The body responds hours to days after the exposure by the release of sensitized T-cells. C) The body responds within minutes to the antigen due to a previous exposure. D) The body makes special autoantibodies directed against self cells.

D. Hypersensitivity reactions are immune responses that are exaggerated or inappropriate against an antigen or allergen. In type II reaction, the body makes special autoantibodies directly against self cells. These self cells have a foreign protein attached to it. These autoantibodies attach to the self cells and form an immune complex. This results in the destruction of the self cells and the attached foreign protein. Examples of type II reaction include: hemolytic anemia, thrombocytopenic purpura, and hemolytic transfusion reaction (receiving the wrong blood type).

What statement best explains the meaning of innate immunity? A) Acquired by the transfer of antibodies from others B) Remembers past exposures and is antigen-specific C) When the body develops an inappropriate immune response D) The body's first line of defense against infection at birth

D. Innate immunity is the body's first line of defense. The job of the innate immunity is to fight to prevent infection. Prior exposure to the antigen is not required for innate immunity to protect the body. This type of immunity is referred to as natural immunity. Innate immunity involves barriers that prevent pathogens from entering the body. These barriers include the cough reflex, mucous, skin, stomach acid, and enzymes in tears and skin oils.

What statement made by the client diagnosed with systemic lupus erythematosus demonstrates a need for further education? A) This is a disease that is chronic and progressive in nature. B) This disease is one that causes my body to attack itself. C) This disease can attack any of the organs in my body. D) This disease is incurable and untreatable by any drugs.

D. Systemic lupus erythematosus (SLE) is a chronic, progressive, inflammatory connective tissue disorder. SLE is considered an autoimmune disorder. It can cause major body organs and systems to fail. Spontaneous remissions and exacerbations occur with SLE that can occur rapidly or slowly. As a type III reaction, immune complexes form causing inflammation and damage. As an autoimmune disorder, there is not a cure for the condition. There is a drug regimen that can help decrease exacerbation events.

A client underwent an antibody test for human immunodeficiency virus (HIV) as part of a screening process and has just been told that the results were positive. Which anticipatory guidance regarding the next step should the nurse provide to the client? A. The client will be started on fluoxetine in 1 month. B. Antiretroviral therapy will begin within 3 months. C. Follow-up testing will be promptly performed to confirm the result. D. The client will be monitored for signs and symptoms of HIV to determine the need for treatment.

ANS: C Rationale: Follow-up testing is performed if the initial test result is positive to ensure a correct diagnosis. These tests include antibody differentiation tests, which distinguish HIV-1 from antibodies, and HIV-1 nucleic acid tests, which look for the virus RNA directly. Antiretroviral therapy may be needed, but the next step would be to confirm the diagnosis. Fluoxetine, an antidepressant, would be prescribed if the client developed severe depression, which is not evident in this scenario. The client would not simply be monitored for signs and symptoms of HIV to determine treatment; the client would undergo follow-up testing to determine the need for treatment.

A nurse is planning the assessment of a client who is exhibiting signs and symptoms of an autoimmune disorder. The nurse should be aware that the incidence and prevalence of autoimmune diseases is known to be higher among which group? A. Young adults B. Native Americans/First Nations C. Women D. People of Hispanic descent

ANS: C Rationale: Many autoimmune diseases have a higher incidence in females than in males, a phenomenon believed to be correlated with sex hormones. Sex hormones play definitive roles in lymphocyte maturation, activation, and synthesis of antibodies and cytokines. Autoimmune disorders in women, such as lupus and multiple sclerosis, may be linked to hormonal changes that can occur during puberty, pregnancy, and menopause. Young adults, Native Americans/First Nations and people of Hispanic descent are not known to have a higher incidence or prevalence of autoimmune disorders.

A nurse should prioritize and closely monitor a client for a potentially severe anaphylactic reaction after the client has received which medical intervention? A. Measles-mumps-rubella vaccine B. Rapid administration of intravenous fluids C. Computed tomography with contrast solution D. Nebulized bronchodilator

ANS: C Rationale: The most severe anaphylaxis, sometimes referred to as anaphylactic shock, is caused by antibiotics and radiocontrast agents. The computed tomography scan with contrast dye uses these agents. Vaccines can produce an anaphylactic reaction but are usually localized and not severe. Intravenous fluid and bronchodilators may be used to manage anaphylaxis in clients with symptoms of bronchospasm or hypotension, but they are not typically associated with triggering anaphylactic shock themselves.

A client is scheduled for a skin test. The client informs the nurse that the client used a corticosteroid earlier today to alleviate allergy symptoms. Which nursing intervention should the nurse implement? A. Note the corticosteroid use in the electronic health record and continue with the test. B. Modify the skin test to check for grass, mold, or dust allergies only. C. Administer sodium valproate to reverse the effects of corticosteroid usage. D. Cancel and reschedule the skin test when the client stops taking the corticosteroid.

ANS: D Rationale: Corticosteroids and antihistamines, including over-the-counter allergy medications, suppress skin test reactivity and should be stopped 48 to 96 hours before testing, depending on the duration of their activity. If the client takes one of these medications within this time frame, the nurse should cancel the skin test and reschedule for a time when the client is not taking it. The nurse should not continue with the test. The nurse should not modify the test. Administration of sodium valproate is used to reverse corticosteroid-induced mania, not to reverse it effects, in general.

A nurse is preparing a client for allergy skin testing. What precautionary step is most important for the nurse to follow? A. The client must not have received an immunization within 7 days. B. The nurse should administer albuterol 30 to 45 minutes prior to the test. C. Prophylactic epinephrine should be given before the test. D. Emergency equipment should be readily available.

ANS: D Rationale: Emergency equipment must be readily available during testing to treat anaphylaxis. Immunizations do not contraindicate testing. Neither epinephrine nor albuterol is given prior to testing.

A client has come into the free clinic asking to be tested for human immunodeficiency virus (HIV) infection. The client asks the nurse how the test works. The nurse responds that if the testing shows that antibodies to the acquired immunodeficiency syndrome (AIDS) virus are present in the blood, this indicates that the client has which of the following? A. Immunity to HIV B. An intact immune system C. An AIDS-related complication D. An HIV infection

ANS: D Rationale: Positive test results indicate that antibodies to the AIDS virus are present in the blood. The presence of antibodies does not imply an intact immune system or specific immunity to HIV. This finding does not indicate the presence of AIDS-related complications.

A nurse is participating in a code blue response to a patient who lost consciousness after a short period of distress. The physician leading the code has ordered the administration of IV epinephrine. When preparing this drug for administration, the nurse should recognize what goal of this treatment? A) Increased cardiac contractility B) Increased blood flow to the heart and brain C) Increased peripheral blood circulation D) Increased release of dopamine

Ans: B Feedback: Epinephrine is often administered during cardiopulmonary resuscitation (CPR). The most important action of epinephrine during cardiac arrest is constriction of peripheral blood vessels, which shunts blood to the central circulation and increases blood flow to the heart and brain. The goal of epinephrine administration is not increased contractility or dopamine release.

A patient is taking ibuprofen (Motrin) for knee pain. The patient is admitted to the hospital with abdominal pain. Which of the following assessments should the nurse prioritize? A) Assessment for diarrhea B) Assessment for occult blood in the patient's stool C) Assessment of the patient's urine for hematuria D) Assessment for hemoptysis

Ans: B Feedback: Nonsteroidal anti-inflammatory agents that block COX-1 and COX-2 place the patient at risk for gastrointestinal bleed. Patients who have symptoms of abdominal pain and are taking NSAIDs should be assessed for signs and symptoms of gastrointestinal bleed. Assessing the patient for diarrhea is not related to ibuprofen (Motrin) administration.Assessing the patient for hematuria or hemoptysis is not a priority.

A patient is administered a nucleotide reverse transcriptase inhibitor in combination with a non nucleotide reverse transcriptase inhibitor. What is the main rationale for administering these medications together? A) They facilitate increased adherence to treatment. B) They decrease the length of illness. C) They have synergistic antiviral effects. D) They prevent the development of opportunistic infections.

Ans: C Feedback: Because the two types of drugs inhibit reverse transcriptase by different mechanisms, they may have synergistic antiviral effects. The use of two medications may decrease adherence because of the burden of taking two medications. The use of two medications will not decrease the length of the illness. They will not decrease all opportunistic infections.

An adult patient has complained to the nurse that she has been experiencing a dry mouth and urinary retention after several nights of taking an OTC sleep aid. The nurse should suspect that this medication contains what antihistamine? A) Loratadine (Claritin) B) Promethazine (Phenergan) C) Diphenhydramine (Benadryl) D) Olopatadine (Patanol)

Ans: C Feedback: The active ingredient in OTC sleep aids is a sedating antihistamine, usually diphenhydramine (Benadryl).

A teenage boy has been brought to the emergency department (ED) by his football coach, who states that the boy has not responded to his normal rescue inhaler. The care team has opted for the administration of subcutaneous epinephrine. The ED nurse should anticipate what assessment finding subsequent to the administration of this drug? A) Increased level of consciousness within 2 to 5 minutes B) Immediate bronchodilation C) An immediate decrease in respiratory rate D) Bronchodilation within 5 to 10 minutes

Ans: D Feedback: For acute asthma attacks, subcutaneous (Sub-Q) administration of epinephrine usually produces bronchodilation within 5 to 10 minutes; maximal effects may occur within 20 minutes. An increase in LOC is not a priority, and respiratory rate would not likely decrease.

A patient with an autoimmune disorder says, I don't know why this happened to me. I try to exercise and eat well. How should the nurse respond? 1. These disorders are usually associated with a vitamin deficiency. 2. These problems happen when your body misinterprets normal cells as being foreign and attempts to destroy them. 3. It happened because you were exposed to something repeatedly, and then the body decided it needed to destroy it. 4. Chronic illnesses are the cause of autoimmune disorders.

Answer: 2 Explanation: 1. Autoimmune disorders are not specifically linked to vitamin deficiencies. 2. One theory about autoimmunity is that of molecular mimicry. This is when the body will react appropriately to an allergen but then incorrectly identifies normal body tissue as being the same allergen and begins to destroy normal tissue. 3. Autoimmune disorders do not occur in response to repeated exposure to an allergen. 4. Autoimmune disorders are not linked specifically to chronic illnesses.

An adult patient is demonstrating anaphylaxis from an insect sting. What is the nurse's priority intervention? 1. Benadryl (diphenhydramine) 50 mg intravenously 2. Oxygen at 3 liters via nasal cannula 3. Epinephrine 1:1000 0.5 mg sq 4. Normal saline at 150 mL/hr

Answer: 3 Explanation: 1. Administration of diphenhydramine is appropriate but is not the initial therapy. 2. Oxygen is administered according to pulse oximetry readings. 3. The patient in anaphylaxis experiences bronchial spasm and constriction. The administration of epinephrine is necessary to reverse this process and facilitate an open airway. This is the priority intervention. 4. After experiencing anaphylaxis, the patient will likely be hospitalized and given IV fluids. This is not the immediate priority.

A female patient is concerned after learning that a person with whom she had a casual sexual encounter has been diagnosed as being HIV positive. Which other patient statement would the nurse evaluate as significant? 1. I have not felt bad since the possible exposure. 2. We were only together for about a week and had sex 3 or 4 times. 3. I did have a cold and sore throat last week, but it has cleared up without problems. 4. I had a normal period just a few days after we broke up.

Answer: 3 Explanation: 1. There is a clinical latency period or asymptomatic stage that is generally present at the beginning of infection. The fact that the patient has not been symptomatic is not significant. 2. The number of exposures is not significant in that infection can occur with one exposure. 3. Within about 2 to 4 weeks after exposure to the virus, a transient flu-like or mononucleosis-like disease may occur. 4. The presence of normal menses does not decrease the risk of infection.

A patient being treated with isoniazid for tuberculosis develops symptoms of systemic lupus erythematosus (SLE). The patient says, I can't believe that I am so sick. First I get TB and now this. What is going to happen to me? What nursing response is indicated? 1. You will have to learn to manage both the TB and the SLE. 2. Once your TB is cured, we can help you fight the SLE. 3. Often the SLE symptoms go away after the TB medication is changed. 4. Your immune system must be under a great deal of stress for both of these diseases to develop.

Answer: 3 Explanation: 1. This is not a therapeutic response and should not be used with this patient. 2. There is no indication that SLE treatment must be delayed until the TB is cured. 3. Drug-induced SLE often resolves upon discontinuation of the drug. 4. Immunity is not associated with the development of this patient's SLE.

A patient receiving a blood transfusion reports nausea and back pain 10 minutes into the transfusion. The nurse realizes the patient is experiencing which type of hypersensitivity response? 1. Type I 2. Type III 3. Type IV 4. Type II

Answer: 4 Explanation: 1. A type I hypersensitivity response occurs after repeated exposure to an allergen that causes an allergen-antigen response. 2. A type III hypersensitivity response is also an allergen-antigen response; however, the complexes are found in tissues. Organ rejection is an example of this type of response. 3. A type IV hypersensitivity response is a delayed response seen after an insect bite or with poison ivy. 4. A hemolytic transfusion reaction is a major example of a type II hypersensitivity response. The reaction will occur within minutes of beginning the transfusion and is an emergency.

Which response provides correct information regarding acquired immunity? Select all that apply. A) The action of the natural killer cells B) Immunity is acquired actively by exposure C) Immunity occurs by passage of antibodies from one host to another D) The action involves T-cell responses E) The action involves B-cell responses

B, C, D, E. Acquired immunity requires prior exposure to an antigen. After the initial exposure, any future exposures will result in the immune system remembering the antigen and attacking the invader. The two methods of developing acquired immunity include adaptive acquired and passive acquired. With adaptive acquired immunity, immunity is acquired actively by exposure to the antigen. With passive acquired immunity, immunity occurs by the passage of antibodies from one host to another. Acquired immunity involves cell-mediated immunity which is derived from T-cell responses and humoral immunity which is derived from B-cell responses.

During a routine health exam, the client informs the nurse that she has been very fatigued in the past 2 weeks. The client states that every morning for the past week, the joints in her hands are very painful, swollen, and stiff. The nurse assesses the client's joints in her hands and notes the redness, swelling, and stiffness. What non-pharmacological nursing interventions would you recommend to help alleviate the swelling, pain, and stiffness? Select all that apply. A) Instruct to perform active ROM exercises every 2 hours B) Instruct to take a warm shower or bath in the morning and before bedtime C) Instruct to alternate heat and cold therapy to affected joints D) Instruct to make an appointment for an hour massage to the affected joints E) Instruct to rest the affected joints when in severe pain

B, C, E. When a client is experiencing a flare-up of rheumatoid arthritis, the client should be instructed on the non-pharmacological nursing interventions to perform before adding medications. The non-pharmacological nursing interventions should include instructing to take a shower or bath in the morning and at night before bedtime. The heat will help reduce the swelling and stiffness and reduce the pain. The client should be instructed to alternate heat and cold therapy to reduce inflammation and alleviate pain. During flare-ups, the client should be instructed to rest the affected joints. Instructing the client to perform active ROM exercises is important to help maintain function and strength, but should not be instructed to do so every 2 hours during a flare-up. The client should be instructed to perform low-impact exercises to assist and maintain function and strength. An hour massage would not be recommended due to increasing the client's pain with the massage.

Which education point will the registered nurse (RN) provide the client when teaching how to administer an epipen? Select all that apply. A) Remove the clothing and clean the skin with an alcohol pad B) Remove the needle cap and insert the needle into the skin C) Inject the drug at a 90 degree angle into the outer thigh muscle area D) Push firmly on the syringe plunger to inject the epinephrine drug E) Repeat this drug every 30 minutes until the symptoms disappear

B, C. Epinephrine is a sympathomimetic drug. This drug stimulates alpha and beta receptors, resulting in vasoconstriction. For anaphylaxis, the drug is administered by an epinephrine auto-injector, also known as an 'epi-pen'. Epinephrine is the priority drug of treatment for an anaphylactic event. This drug is administered intramuscularly and has a rapid onset, but a 10 to 20 minute duration. For the emergency administration of this drug, skin prep is not indicated. The drug is administered into the outer muscle of the thigh at a 90 degree angle. Once inserted into the muscle, the syringe is held in place for 10 seconds allowing the drug/syringe to self-inject. The drug is repeated every 5 to 15 minutes until all anaphylactic symptoms have resolved. Emergency care should be sought immediately after the administration of the epi-pen as the drug has a short duration. This drug is stored at room temperature, in a dark location. Expected side-effects of this drug include: dizziness, palpitations, and tachycardia.

Which of these clients are exhibiting the signs and symptoms associated with rheumatoid arthritis? Select all that apply. A) A 30-year-old football star with crepitus in the left knee B) A 35-year-old client with severe warmth, redness, and inflammation of both hands C) A 40-year-old female with fatigue, joint stiffness, and joint pain in the mornings D) A 32-year-old ice skater with soreness and swelling in both ankles E) A 42-year-old male with a grating sensation and MRI results revealing bone spur formation in the right knee

B, C. Rheumatoid arthritis signs and symptoms include fatigue, anorexia, weight loss, morning joint stiffness, symmetrical joint swelling, and severe pain in the small joints of the hands. Rheumatoid arthritis causes swans neck, boutonniere's deformity, contractures of the joints, joint pain that gets better with activity and gets worse at rest. Rheumatoid arthritis results in deconditioning of the joints resulting in decreased muscle strength with systemic symptoms. Currently, no cure exists for rheumatoid arthritis. health care provider (HCP)s teach management of fatigue, joint pain and stiffness through low impact exercises, range-of-motion exercises, and alternating heat and cold applications.

A client presents at the clinic with an allergic disorder. The client asks the nurse what an allergic disorder means. What would be the nurse's best response? A. It means you are very sensitive to something inside of yourself. B. It is a hyperimmune response to something in the environment that is usually harmless. C. It is a harmless reaction to something in the environment. D. It is a muted response to something in the environment.

B. An allergic disorder is characterized by a hyperimmune response to weak antigens that usually are harmless. The antigens that can cause an allergic response are called allergens.

The nurse practitioner who is monitoring the patient's progression of HIV is aware that the most debilitating gastrointestinal condition found in up to 90% of all AIDS patients is: A. Anorexia. B. Chronic diarrhea. C. Nausea and vomiting. D. Oral candida.

B. Chronic diarrhea is believed related to the direct effect of HIV on cells lining the intestine. Although all gastrointestinal manifestations of AIDS can be debilitating, the most devastating is chronic diarrhea. It can cause profound weight loss and severe fluid and electrolyte imbalances.

An adult client has had mumps when the client was a child. The client had a titer prior to entering nursing school and shows immunity. What type of immunity does this reflect? A. Passive immunity B. Naturally acquired active immunity C. Artificially acquired active immunity D. Natural passive immunity

B. Naturally acquired active immunity occurs as a direct result of infection by a specific microorganism. An example is the immunity to measles that develops after the initial infection. Not all invading microorganisms produce a response that gives lifelong immunity. Artificially acquired immunity is obtained by receiving a killed or weakened microorganism or toxoid. Passive immunity is acquired when ready-made antibodies are given to a susceptible person.

A laboring mother asks the nurse if the baby will have immunity to some illnesses when born. What type of immunity does the nurse understand that the newborn will have? A. Naturally acquired active immunity B. Passive immunity transferred by the mother C. Artificially acquired active immunity D. There is no immunity passed down from mother to child.

B. Passive immunity develops when ready-made antibodies are given to a susceptible person. The antibodies provide immediate but short-lived protection from the invading antigen. Newborns receive passive immunity to some diseases for which their mothers have manufactured antibodies. Naturally acquired active immunity occurs as a direct result of infection by a specific microorganism. An example is the immunity to measles that develops after the initial infection. Not all invading microorganisms produce a response that gives lifelong immunity. Artificially acquired immunity is obtained by receiving a killed or weakened microorganism or toxoid.

Which intervention is the single most important aspect for the client at risk for anaphylaxis? A. Use of antihistamines B. Prevention C. Desensitization D. Wearing a medical alert bracelet

B. Prevention involves strict avoidance of potential allergens for the individual at risk for anaphylaxis. If avoidance of or exposure to allergens is impossible then the individual should be prepared with an emergency kit containing epinephrine for injection to prevent the onset of the reaction upon exposure. While helpful, there must be no lapses in desensitization therapy because this may lead to the reappearance of an allergic reaction when the medication is reinstituted. A medical alert bracelet will assist those rendering aid to a client who has experienced an anaphylactic reaction. antihistamines may not be effective in preventing anaphylaxis.

The nurse is teaching a newly diagnosed client about systemic lupus erythematosus(SLE). What statement by the client indicates the teaching was successful? A. The symptoms are primarily localized to the skin but may involve the joints. B. The belief is that it is an autoimmune disorder with an unknown trigger. C. SLE has very specific manifestations that make diagnosis relatively easy. D. This disorder is more common in men in their thirties and forties than in women.

B. Systemic lupus erythematosus is believed to be an autoimmune disorder but the triggering mechanism is not known. The disorder is more common in women than in men, most with the disorder in the 3rd or 4th decade of life. The disease is considered the great imitator because the clinical signs resemble many other conditions. SLE is a diffuse connective tissue disease that affects multiple body systems.

The client is about to have a skin test for an allergic disorder. What critical instruction should the nurse give this client? A. Avoid red meat for 48 to 72 hours before the test. B. Avoid antihistamines and cold preparations for 48 to 72 hours before the test. C. Avoid sunlight for 48 to 72 hours before the test. D. Avoid strenuous physical activity for 24 hours before the test.

B. The nurse should instruct the client to avoid taking prescribed or over-the-counter antihistamine or cold preparations for at least 48 to 72 hours before a skin test because this reduces the potential for false-negative test results. The nurse should not ask the client to avoid red meat, strenuous physical activity, or sunlight because these do not affect the test results.

A client that is HIV+ has been diagnosed with Pneumocystis pneumonia caused by P. jiroveci. What medication does the nurse expect that the client will take for the treatment of this infection? A. Nystatin B. Trimethoprim-sulfamethoxazole C. Fluconazole D. Amphotericin B

B. To prevent and treat Pneumocystis pneumonia, trimethoprim-sulfamethoxazole (Bactrim, Septra) is prescribed. The other medications are antifungals and used to treat candidiasis.

Which clinical manifestation is most common in a client diagnosed with systemic lupus erythematosus (SLE)? Select all that apply. A) Pruritus and burning skin pain B) Presence of vesicles and bullae C) Butterfly shaped facial rash D) Fever over 100 F E) Swollen, painful joints

C, D, E. Systemic lupus erythematosus (SLE) is thought to be an autoimmune process. Clinical findings vary between clients. The most characteristic clinical manifestation is a butterfly shaped rash in the cheeks and nose. Other common clinical manifestations include swollen, pain joints and a fever greater than 100 F (37.7 C). Manifestations referable to any organ system may appear. Periodic exacerbations may occur.

When assisting the patient to interpret a negative HIV test result, the nurse informs the patient that the results mean which of the following? A. He is immune to HIV. B. He has not been infected with HIV. C. Antibodies to HIV are not present in his blood. D. Antibodies to HIV are present in his blood.

C. A negative test result indicates that antibodies to HIV are not present in the blood at the time the blood sample for the test is drawn. A negative test result should be interpreted as demonstrating that if infected, the body has not produced antibodies (which take from 3 weeks to 6 months or longer). Therefore, subsequent testing of an at-risk patient must be encouraged. The test result does not mean that the patient is immune to the virus, nor does it mean that the patient is not infected. It just means that the body may not have produced antibodies yet. When antibodies to HIV are detected in the blood, the test is interpreted as positive.

A patient asks the nurse if it would be all right to take an over-the-counter antihistamine for the treatment of a rash. What should the nurse educate the patient is a major side effect of antihistamines? A. Diarrhea B. Palpitations C. Sedation D. Anorexia

C. Antihistamines are the major class of medications prescribed for the symptomatic relief of allergic rhinitis. The major side effect is sedation, although H1 antagonists are less sedating than earlier antihistamines.

In its attempt to suppress allergic responses, the body releases several chemicals which have a role in mediating physical reactions. Epinephrine, which interferes with vasoactive chemical release from mast cells, is instrumental in suppressing which type of hypersensitivity response? A. type IV B. type III C. type I D. type II

C. Epinephrine interferes with the release of vasoactive chemicals from mast cells which cause vasodilation during anaphylaxis, also known as a Type I response.

A client with rheumatoid arthritis calls the clinic for an emergency visit. When the client arrived, she stated that she had sudden blurred vision changes. Which medication on the client's list causes sudden retinal changes? A) Methotrexate B) Sulfasalazine C) Hydroxychloroquine D) Calcitonin

C. Hydroxychloroquine is a disease-modifying antirheumatic drug (DMARD) used in the treatment of rheumatoid arthritis. A major side-effect of the drug is sudden visual changes or retinopathy that may be permanent. The client should schedule a routine eye exam after starting the medication to get a baseline of the eyes. Then, the client should schedule an eye exam every 6 months while taking the medication. Hydroxychloroquine can also cause the client's insulin level to increase and the client may experience severe hypoglycemia. This medication can cause QT interval prolongation if the client has a history of renal disease.

A client with rheumatoid arthritis tells the nurse, I know it is important to exercise my joints so that I won't lose mobility, but my joints are so stiff and painful that exercising is difficult. Which of the following responses by the nurse would be most appropriate? 1. You are probably exercising too much. Decrease your exercise to every other day. 2. Tell the physician about your symptoms. Maybe your analgesic medication can be increased. 3. Stiffness and pain are part of the disease. Learn to cope by focusing on activities you enjoy. 4. Take a warm tub bath or shower before exercising. This may help with your discomfort.

4. Superficial heat applications, such as tub baths, showers, and warm compresses, can be helpful in relieving pain and stiffness. Exercises can be performed more comfortably and more effectively after heat applications. The client with rheumatoid arthritis must balance rest with exercise every day, not every other day. Typically, large doses of analgesics, which can lead to hepatotoxic effects, are not necessary. Learning to cope with the pain by refocusing is inappropriate.

The community health nurse is conducting a research study and is identifying clients in the community at risk for latex allergy. Which client population is most at risk for developing this type of allergy? A) Hairdressers B) Unhoused people C) Children in day care centers D) Individuals living in a group home

A

A client is experiencing an acute exacerbation of rheumatoid arthritis. What should the nursing priority be? A. Administering ordered analgesics and monitoring their effects B. Performing meticulous skin care C. Providing comprehensive client teaching; including symptoms of the disorder, treatment options, and expected outcomes D. Supplying adaptive devices, such as a zipper-pull, easy-to-open beverage cartons, lightweight cups, and unpackaged silverware

A. An acute exacerbation of rheumatoid arthritis can be very painful, and the nurse should make pain management her priority. Client teaching, skin care, and supplying adaptive devices are important, but these actions don't not take priority over pain management.

Which assessment suggests to the nurse that a client with systemic lupus erythematosus is having renal involvement? A. Hypertension B. Decreased cognitive ability C. Behavioral changes D. Chest pain

A. Hypertension is suggestive of renal damage in the client with systemic lupus erythematosus.

A nurse's plan of care for a client with rheumatoid arthritis includes several exercise-based interventions. What goal should the nurse prioritize? A. Maximize range of motion while minimizing exertion. B. Increase joint size and strength. C. Limit energy output in order to preserve strength for healing. D. Preserve or increase range of motion while limiting joint stress.

ANS: D Rationale: Exercise is vital to the management of rheumatic disorders. Goals should be preserving and promoting mobility and joint function while limiting stress on the joint and possible damage. Cardiovascular exertion should remain within age-based limits and individual ability, but it is not a goal to minimize exertion. Increasing joint size is not a valid goal.

A neonate exhibited some preliminary signs of infection, but the infant's condition resolved spontaneously prior to discharge home from the hospital. This infant's recovery was most likely due to which type of immunity? A. Cytokine immunity B. Specific immunity C. Active acquired immunity D. Nonspecific immunity

ANS: D Rationale: Natural immunity, or nonspecific immunity, is present at birth. Active acquired or specific immunity develops after birth. Cytokines are proteins that mediate the immune response; they are not a type of immunity.

A client calls the nurse in the emergency department and reports being just stung by a bee while gardening. The client is afraid of a severe reaction because the client's neighbor experienced such a reaction a week ago. Which action would the nurse take? A) Advise the client to soak the site in hydrogen peroxide B) Ask the client if they've ever sustained a bee sting in the past C) Tell the client to call an ambulance for transport to the emergency department D) Tell the client not to worry about the sting unless difficulty with breathing occurs

B

A client suspected of having systemic lupus erythematosus (SLE) is being scheduled for testing. The client asks which of the tests ordered will determine positivity for the disorder. Which statement by the nurse is most accurate? A. You should discuss that matter with your healthcare provider. B. Tell me more about your concerns about this potential diagnosis. C. The diagnosis won't be based on the findings of a single test but by combining all data found. D. SLE is a very serious systemic disorder.

C. There is no single test available to diagnose SLE. Therefore, the nurse should inform the client that diagnosis is based on combining the findings from the physical assessment and the laboratory tests results. Advising the client to speak with the health care provider, stating that SLE is a serious systemic disorder, and asking the client to express feelings about the potential diagnosis do not answer the client's question.

A client with rheumatoid arthritis arrives at the clinic for a checkup. Which statement by the client refers to the most overt clinical manifestation of rheumatoid arthritis? A. I have pain in my hands. B. I have trouble with my balance. C. My legs feel weak. D. My finger joints are oddly shaped.

D. Joint abnormalities are the most obvious manifestations of rheumatoid arthritis. A systemic disease, rheumatoid arthritis attacks all connective tissue. Although muscle weakness may occur from limited use of the joint where the muscle attaches, such weakness isn't the most obvious sign of rheumatoid arthritis; also, it occurs only after joint abnormalities arise. Subcutaneous nodules in the hands, although common in rheumatoid arthritis, are painless. The disease may cause gait disturbances, but these follow joint abnormalities.

Which of the following disorders is characterized by an increased autoantibody production? A. Scleroderma B. Rheumatoid arthritis (RA) C. Polymyalgia rheumatic D. Systemic lupus erythematosus (SLE)

D. SLE is an immunoregulatory disturbance that results in increased autoantibody production. Scleroderma occurs initially in the skin but also occurs in blood vessels, major organs, and body systems, potentially resulting in death. Rheumatoid arthritis results from an autoimmune response in the synovial tissue, with damage taking place in body joints. In polymyalgia rheumatic, immunoglobulin is deposited in the walls of inflamed temporal arteries.

A nurse is administering a mumps vaccine to an adolescent. Which of the following medications should be available when administering an immunization? A) Diphenhydramine (Benadryl) B) Hydroxyzine (Vistaril) C) Physostigmine D) Epinephrine

D. The administration of vaccines for immunization possesses the risk of an allergic reaction and anaphylaxis. The nurse should have aqueous epinephrine available in the event of an anaphylactic reaction. The administration of diphenhydramine or hydroxyzine will reduce the allergic reaction but will not be effective in the event of anaphylaxis. Physostigmine is not administered.

The nurse knows the best strategy for latex allergy is A. antihistamines. B. corticosteroids. C. epinephrine from an emergency kit. D. avoidance of latex-based products.

D. The best strategy available for latex allergy is to avoid latex-based products, but this is often difficult because of their widespread use. Antihistamines and an emergency kit containing epinephrine should be provided to these clients, along with instructions about emergency management of latex allergy.

Which disorder is characterized by a butterfly-shaped rash across the bridge of the nose and cheeks? A. Scleroderma B. Rheumatoid arthritis (RA) C. Polymyositis D. Systemic lupus erythematosus (SLE)

D. The most familiar manifestation of SLE is an acute cutaneous lesion consisting of a butterfly-shaped rash across the bridge of the nose and the cheeks. This type of rash does not characterize RA, scleroderma, or polymyositis.

Which of these clients are exhibiting the signs and symptoms associated with rheumatoid arthritis? Select all that apply. A) A 30-year-old football star with crepitus in the left knee B) A 35-year-old client with severe warmth, reedness, and inflammation of both hands C) A 40-year-old female with fatigue, joint stiffness, and joint pain in the mornings D) A 32-year-old ice skater with soreness and swelling in both ankles E) A 42-year-old male with a grating sensation and MRI results revealing bone spur formation in the right knee

B, C. Rheumatoid arthritis signs and symptoms include fatigue, anorexia, weight loss, morning joint stiffness, symmetrical joint swelling, and severe pain in the small joints of the hands. Rheumatoid arthritis causes swans neck, boutonniere's deformity, contractures of the joints, joint pain that gets better with activity and gets worse at rest. Rheumatoid arthritis results in deconditioning of the joints resulting in decreased muscle strength with systemic symptoms. Currently, no cure exists for rheumatoid arthritis. health care provider (HCP)s teach management of fatigue, joint pain and stiffness through low impact exercises, range-of-motion exercises, and alternating heat and cold applications.

Which of the following procedures involves a surgical fusion of the joint? A. Osteotomy B. Arthrodesis C. Tenorrhaphy D. Synovectomy

B. An arthrodesis is a surgical fusion of the joint. Synovectomy is the excision of the synovial membrane. Tenorrhaphy is the suturing of a tendon. An osteotomy alters the distribution of the weight within the joint.

Kaposi sarcoma (KS) is diagnosed through A. visual assessment. B. computed tomography. C. biopsy. D. skin scraping.

C. KS is diagnosed through biopsy of the suspected lesions. Visual assessment will not confirm a diagnosis. A computed tomography scan will not assist in determining skin cell changes. Skin scraping is a procedure to collect cells, not to evaluate cells.

An elderly patient has taken ibuprofen (Motrin) 800 mg two times per day for the past 3 years. Which of the following laboratory tests is the priority assessment? A) Renin and aldosterone levels B) 24-hour urine for microalbumin C) Blood urea nitrogen and serum creatinine D) Complete blood count

C. Nonsteroidal anti-inflammatory agents in long-term use can cause renal impairment. The patient should be assessed for renal impairment with the elevation of the serum BUN and creatinine. NSAIDs do not affect renin and aldosterone levels. A 24-hour urine for microalbumin is not recommended when administering ibuprofen. A complete blood count may not be necessary.

Which statement best explains the goal of HAART (highly active antiretroviral therapy) when used for HIV/AIDS? A) The goal of HAART is to decrease the viral load by restricting CD4 cells B) The purpose of HAART is to activate immune cells that kill infected cells and result in viral replication (that is CD8 cells or cytotoxic cells.) C) The goal of HAART is to halt the replication of the virus to prohibit the increase in viral load (CD4 cells produce cytokines). D) HAART is used to treat HIV by destroying the cells in the body, the good with the bad, in order to keep the virus from spreading within.

C. The goal of HAART is to decrease the viral load by stopping the virus from replicating. HAART utilizes several different classes of drugs that prevent the virus from entering the body's CD4 cells. It also disables the proteins required for the virus to replicate. This results in decreasing the viral load and restoring the CD4 level to help immune function. HAART therapy reduces the complications from HIV, as well as transmissions to others.

The nurse is caring for a client recently diagnosed with rheumatoid arthritis. The client asks the nurse, Is there a cure for rheumatoid arthritis? Which would be the best response by the nurse? A) Yes, the cure is bedrest and alternating heat and cold applications to the inflamed joints. B) Yes, the cure is to limit exercise. C) No, treatment is aimed at management of symptoms. D) No, no cure exists. Relaxation is the best treatment.

C. Treatment of rheumatoid arthritis is aimed at the management of symptoms such as fatigue, joint pain and stiffness. Low impact exercises such as swimming, riding a stationary bike, and light walking are recommended to help with pain and joint stiffness and deformity. Alternating heat and cold compresses is recommended to help with the pain and swelling. Performing active range-of-motion (ROM) exercises helps maintain strength and reduces deconditioning. Medications such as NSAIDs, steroids, and methotrexate are utilized to treat the symptoms of rheumatoid arthritis.

On a visit to the clinic, a client reports the onset of early symptoms of rheumatoid arthritis. The nurse should conduct a focused assessment for: 1. Limited motion of joints. 2. Deformed joints of the hands. 3. Early morning stiffness. 4. Rheumatoid nodules.

3. Initially, most clients with early symptoms of rheumatoid arthritis complain of early morning stiffness or stiffness after sitting still for a while. Later symptoms of rheumatoid arthritis include limited joint range of motion; deformed joints, especially of the hand; and rheumatoid nodules.

DELETE Which assessment suggests to the nurse that a client with systemic lupus erythematous is having renal involvement? A. Hypertension B. Decreased cognitive ability C. Behavioral changes D. Chest pain

4. Carrying a laundry basket with clenched fingers and fists is not an example of conserving energy of small joints. The laundry basket should be held with both hands opened as wide as possible and with outstretched arms so that pressure is not placed on the small joints of the fingers. When rising from a chair, the palms should be used instead of the fingers so as to distribute weight over the larger area of the palms. Holding packages close to the body provides greater support to the shoulder, elbow, and wrist joints because muscles of the arms and hands are used to stabilize the weight against the body. This decreases the stress and weight or pull on small joints such as the fingers. Objects can be slid with the palm of the hand, which distributes weight over the larger area of the palms instead of stressing the small joints of the fingers to pick up the weight of the object to move it to another place.

The nurse is performing discharge teaching for a client with rheumatoid arthritis. What teachings are priorities for the client? Select all that apply. A. Assistive devices B. Safe exercise C. Dressing changes D. Medication dosages and side effects E. Narcotic safety

A, B, D. The client with rheumatoid arthritis who is being discharged to home needs information on how to exercise safely to maintain joint mobility. Medication doses and side effects are always an essential part of discharge teaching. Assistive devices, such as splints, walkers, and canes, may assist the client to perform safe self-care. Narcotics are not commonly used, and there would be no reason for dressings.

A client has a serum study that is positive for the rheumatoid factor. What will the nurse tell the client about the significance of this test result? A. It is suggestive of rheumatoid arthritis. B. It is diagnostic for systemic lupus erythematosus. C. It is diagnostic for Sjögren's syndrome. D. It is specific for rheumatoid arthritis.

A. Rheumatoid factor is present in about 70% to 80% of patients with rheumatoid arthritis, but its presence alone is not diagnostic of rheumatoid arthritis, and its absence does not rule out the diagnosis. The antinuclear antibody (ANA) test is used to diagnose Sjögren's syndrome and systemic lupus erythematosus.

A client's decreased mobility has been attributed to an autoimmune reaction originating in the synovial tissue, which caused the formation of pannus. This client has been diagnosed with which health problem? A. Rheumatoid arthritis (RA) B. Systemic lupus erythematosus (SLE) C. Osteoporosis D. Polymyositis

ANS: A Rationale: In RA, the autoimmune reaction results in phagocytosis, producing enzymes within the joint that break down collagen, cause edema and proliferation of the synovial membrane, and ultimately form pannus. Pannus destroys cartilage and bone. SLE, osteoporosis, and polymyositis do not involve pannus formation.

A nurse is assessing the skin integrity of a client who has AIDS. When performing this inspection, the nurse should prioritize assessment of what skin surfaces? A. Perianal region and oral mucosa B. Sacral region and lower abdomen C. Scalp and skin over the scapulae D. Axillae and upper thorax

ANS: A Rationale: The nurse should inspect all the client's skin surfaces and mucous membranes, but the oral mucosa and perianal region are particularly vulnerable to skin breakdown and fungal infection.

A nurse is performing the admission assessment of a client who has AIDS. What components should the nurse include in this comprehensive assessment? Select all that apply. A. Current medication regimen B. Identification of client's support system C. Immune system function D. Genetic risk factors for HIV E. History of sexual practices

ANS: A, B, C, E Rationale: Nursing assessment includes numerous focuses, including identification of medication use, support system, immune function, and sexual history. HIV does not have a genetic component.

A nurse is performing the initial assessment of a client who has a recent diagnosis of systemic lupus erythematosus (SLE). Which skin manifestation would the nurse expect to observe on inspection? A. Petechiae B. Erythematous rash C. Jaundice D. Skin sloughing

ANS: B Rationale: An acute cutaneous lesion consisting of an erythematous (butterfly-shaped) rash across the bridge of the nose and cheeks occurs in SLE. Petechiae are pinpoint skin hemorrhages, which are not a clinical manifestation of SLE. Clients with SLE do not typically experience jaundice or skin sloughing.

A nurse is planning client education for a client being discharged home with a diagnosis of rheumatoid arthritis. The client has been prescribed antimalarials for treatment, so the nurse knows to teach the client to self-monitor for what adverse effect? A. Tinnitus B. Visual changes C. Stomatitis D. Hirsutism

ANS: B Rationale: Antimalarials may cause visual changes; regular ophthalmologic examinations are necessary. Tinnitus is associated with salicylate therapy and hirsutism is associated with corticosteroid therapy. Antimalarials do not normally cause stomatitis.

A nurse is providing care for a client who has a rheumatic disorder. The nurse's focused assessment includes the client's mood, behavior, level of consciousness, and neurologic status. Which diagnosis is most likely for this client? A. Osteoarthritis (OA) B. Systemic lupus erythematosus (SLE) C. Rheumatoid arthritis (RA) D. Gout

ANS: B Rationale: SLE has a high degree of neurologic involvement and can result in central nervous system changes. The client and family members are asked about any behavioral changes, including manifestations of neurosis or psychosis. Signs of depression are noted, as are reports of seizures, chorea, or other central nervous system manifestations. OA, RA, and gout lack this dimension.

A client with rheumatoid arthritis comes into the clinic for a routine check-up. On assessment the nurse notes that the client appears to have lost some ability to function since the last office visit. What is the nurse's most appropriate action? A. Arrange a family meeting in order to explore assisted living options. B. Refer the client to a support group. C. Arrange for the client to be assessed in the home environment. D. Refer the client to social work.

ANS: C Rationale: Assessment in the client's home setting can often reveal more meaningful data than an assessment in the health care setting. There is no indication that assisted living is a pressing need or that the client would benefit from social work or a support group.

A client's natural immunity is enhanced by processes that are inherent in the physical and chemical barriers of the body. What is a chemical barrier that enhances natural immunity? A. Cell cytoplasm B. Interstitial fluid C. Gastric secretions D. Cerebrospinal fluid

ANS: C Rationale: Chemical barriers, such as mucus, acidic gastric secretions, enzymes in tears and saliva, and substances in sebaceous and sweat secretions, act in a nonspecific way to destroy invading bacteria and fungi. Not all body fluids are chemical barriers, however. Cell cytoplasm, interstitial fluid, and CSF are not normally categorized as chemical barriers to infection.

A client with systemic lupus erythematosus (SLE) is preparing for discharge. The nurse knows that the client has understood health education when the client makes what statement? A. I'll make sure I get enough exposure to sunlight to keep up my vitamin D levels. B. I'll try to be as physically active as possible between flare-ups. C. I'll make sure to monitor my body temperature on a regular basis. D. I'll stop taking my steroids when I get relief from my symptoms.

ANS: C Rationale: Fever can signal an exacerbation and should be reported to the health care provider. Sunlight and other sources of ultraviolet light may precipitate severe skin reactions and exacerbate the disease. Fatigue can cause a flare-up of SLE. Clients should be encouraged to pace activities and plan rest periods. Corticosteroids must be gradually tapered because they can suppress the function of the adrenal gland. As well, these drugs should not be independently adjusted by the client.

A nurse is performing the health history and physical assessment of a client who has a diagnosis of rheumatoid arthritis (RA). What assessment finding is most consistent with the clinical presentation of RA? A. Cool joints with decreased range of motion B. Signs of systemic infection C. Joint stiffness lasting longer than 1 hour, especially in the morning D. Visible atrophy of the knee and shoulder joints

ANS: C Rationale: In addition to joint pain and swelling, another classic sign of RA is joint stiffness lasting longer than 1 hour, especially in the morning. Joints are typically swollen, not atrophied, and systemic infection does not accompany the disease. Joints are often warm rather than cool.

A client with systemic lupus erythematosus (SLE) asks the nurse why the client has to come to the office so often for check-ups. Which rationale for frequent office visits would be best for the nurse to mention? A. Seeing the client face to face B. Ensuring that the client is taking medications as prescribed C. Monitoring the disease process and how well the prescribed treatment is working D. Drawing blood work every month

ANS: C Rationale: The goals of treatment include preventing progressive loss of organ function, reducing the likelihood of acute disease, minimizing disease-related disabilities, and preventing complications from therapy. Management of SLE involves regular monitoring to assess disease process and therapeutic effectiveness. Stating the benefit of face-to-face interaction does not answer the client's question. Blood work is not necessarily drawn monthly, and assessing medication adherence is not the sole purpose of visits.

A client's rheumatoid arthritis (RA) has failed to respond appreciably to first-line treatments and the primary provider has added prednisone to the client's drug regimen. What principle will guide this aspect of the client's treatment? A. The client will need daily blood testing for the duration of treatment. B. The client must stop all other drugs 72 hours before starting prednisone. C. The drug should be used at the highest dose the client can tolerate. D. The drug should be used for as short a time as possible.

ANS: D Rationale: Corticosteroids are used for shortest duration and at lowest dose possible to minimize adverse effects. Daily blood work is not necessary and the client does not need to stop other drugs prior to using corticosteroids.

The nurse is completing a focused assessment addressing a client's immune function. What should the nurse prioritize in the physical assessment? A. Percussion of the client's abdomen B. Palpation of the client's liver C. Auscultation of the client's apical heart rate D. Palpation of the client's lymph nodes

ANS: D Rationale: During the assessment of immune function, the anterior and posterior cervical, supraclavicular, axillary, and inguinal lymph nodes are palpated for enlargement. If palpable nodes are detected, their location, size, consistency, and reports of tenderness on palpation are noted. Because of the central role of lymph nodes in the immune system, they are prioritized over the heart, liver, and abdomen, even though these would be assessed.

A nurse is reviewing the immune system before planning an immunocompromised client's care. How should the nurse characterize the humoral immune response? A. Specialized cells recognize and ingest cells that are recognized as foreign. B. T lymphocytes are assisted by cytokines to fight infection. C. Lymphocytes are stimulated to become cells that attack microbes directly. D. Antibodies are made by B lymphocytes in response to a specific antigen.

ANS: D Rationale: The humoral response is characterized by the production of antibodies by B lymphocytes in response to a specific antigen. Phagocytosis and direct attack on microbes occur in the context of the cellular immune response.

A patient began antiretroviral therapy several weeks ago for the treatment of HIV, and he has now presented to the clinic for a scheduled follow-up appointment. He states of the nurse, I've been pretty good about taking all my pills on time, though it was a bit hit and miss over the holiday weekend. How should the nurse best respond to this patient's statement? A) Remember that if you miss a dose, you need to take a double dose at the next scheduled time. B) It's acceptable to miss an occasional dose as long as your symptoms don't get worse, but it's not really recommended. C) Remember that your antiretroviral drugs will only be effective if you take them very consistently. D) If you're not consistent with taking your medications, you're likely to develop more side effects.

Ans: C Feedback: Effective treatment of HIV infection requires close adherence to drug therapy regimens involving several drugs and daily doses. Missing as few as one or two doses can decrease blood levels of antiretroviral drugs and result in increased HIV replication and development of drug-resistant viral strains.

The pathophysiology of systemic lupus erythematosus (SLE) is characterized by: A. Destruction of nucleic acids and other self-proteins by autoantibodies B. Overproduction of collagen that disrupts the functioning of internal organs C. Formation of abnormal IgG that attaches to cellular antigens, activating complement D. Increased activity of T-suppressor cells with B-cell hypoactivity, resulting in an immunodeficiency

Answer: A Destruction of nucleic acids and other self-proteins by autoantibodies

A patient with newly diagnosed SLE asks the nurse how the disease will affect her life. The best response by the nurse is: A. You can plan to have a near-normal life since SLE rarely causes death B. It is difficult to tell because to disease is so variable in its severity and progression C. Lifespan is shortened somewhat in people with SLE, but the disease can be controlled with long-term use of corticosteroids D. Most people with SLE have alternating periods of remissions and exacerbations with rapid progression to permanent organ damage

Answer: B It is difficult to tell because to disease is so variable in its severity and progression

Following instruction for a patient with newly diagnosed systemic lupus erythematosus (SLE), the nurse determines that teaching about the disease has been effective when the patient says: A. I should expect to have a low fever all the time with this disease. B. I need to restrict my exposure to sunlight to prevent an acute onset of symptoms. C. I should try to ignore my symptoms as much as possible and have a positive outlook. D. I can expect a temporary improvement in my symptoms if I become pregnant.

Answer: B Rationale: Sun exposure is associated with SLE exacerbation, and patients should use sunscreen with an SPF of at least 15 and stay out of the sun between 11:00 AM and 3:00 PM. Low-grade fever may occur with an exacerbation but should not be expected all the time. A positive attitude may decrease the incidence of SLE exacerbations, but patients are taught to self-monitor for symptoms that might indicate changes in the disease process. Symptoms may worsen during pregnancy and especially during the postpartum period.

A patient with polyarthralgia with joint swelling and pain is being evaluated for systemic lupus erythematosus (SLE). The nurse knows that the serum test result that is the most specific for SLE is the presence of: A. Rheumatoid factor. B. Anti-Smith antibody (Anti-Sm). C. Antinuclear antibody (ANA). D. Lupus erythematosus (LE) cell prep.

Answer: B Rationale: The anti-Sm is antibody found almost exclusively in SLE. The other blood tests are also used in screening but are not as specific to SLE.

The nurse monitors a patient to have Systemic Lupus Erythematosus. Which of the following symptoms is characteristic of this diagnosis? A. Increased T-cell count B. Scaley, inflamed rash on shoulders, neck, and face C. Swelling of the extremities D. Decreased erythrocyte sedimentation rate (ESR)

Answer: B Scaley, inflamed rash on shoulders, neck, and face

A client is suspected of having systemic lupus erythematosus. The nurse monitors the client, knowing that which of the following is one of the initial characteristic sign of systemic lupus erythematosus? A. Weight gain B. Subnormal temperature C. Elevated red blood cell count D. Rash on the face across the bridge of the nose

Answer: D Rationale: Skin lesions or rash on the face across the bridge of the nose and on the cheeks is an initial characteristic sign of systemic lupus erythematosus (SLE). Fever and weight loss may also occur. Anemia is most likely to occur later in SLE.

The nurse is talking with a group of teens about transmission of human immunodeficiency virus (HIV). What body fluids does the nurse inform them will transmit the virus? Select all that apply. A. urine B. semen C. vaginal secretions D. blood E. breast milk

B, C, D, E. There are only four known body fluids through which HIV is transmitted: blood, semen, vaginal secretions, and breast milk. HIV may be present in saliva, tears, and conjunctival secretions, but transmission of HIV through these fluids has not been implicated. HIV is not found in urine, stool, vomit, or sweat.

A nurse is preparing a client with systemic lupus erythematosus (SLE) for discharge. Which instruction should the nurse include in the teaching plan? A. There are no activity limitations between flare-ups. B. Monitor your body temperature. C. Exposure to sunlight will help control skin rashes. D. Corticosteroids may be stopped when symptoms are relieved.

B. The nurse should instruct the client to monitor body temperature. Fever can signal an exacerbation and should be reported to the physician. Sunlight and other sources of ultraviolet light may precipitate severe skin reactions and exacerbate the disease. Fatigue can cause a flare-up of SLE. Clients should be encouraged to pace activities and plan rest periods. Corticosteroids must be gradually tapered because they can suppress the function of the adrenal gland. Abruptly stopping corticosteroids can cause adrenal insufficiency, a potentially life-threatening situation.

A client is experiencing symptoms of rheumatoid arthritis. Which laboratory tests will the nurse expect to be prescribed for this client? Select all that apply. A. Hematocrit B. Creatinine C. Rheumatoid factor D. Erythrocyte count E. Antinuclear antibody

C, D, E. Various blood studies can be done to help diagnose rheumatic diseases. Erythrocyte count may be decreased in rheumatoid arthritis. Rheumatoid factor is present in 80% of those with rheumatoid arthritis. A positive antinuclear antibody test may be associated with rheumatoid arthritis. Creatinine and hematocrit are not used to diagnose rheumatoid arthritis.

A client is being discharged from the hospital after being diagnosed with and treated for systemic lupus erythematosus (SLE). What would the nurse not say when teaching the client and family information about managing the disease? A. Avoid sunlight and ultraviolet radiation. B. Maintain a well-balanced diet and increase fluid intake to raise energy levels and promote tissue healing. C. If you have problems with a medication, you may stop it until your next physician visit. D. Pace activities.

C. Take medications exactly as directed and do not stop the medication if symptoms are relieved unless advised to do so by the physician. Sunlight tends to exacerbate the disease. Because fatigue is a major issue, allow for adequate rest, along with regular activity to promote mobility and prevent joint stiffness. Maintain a well-balanced diet and increase fluid intake to raise energy levels and promote tissue healing.

During a routine checkup, a nurse assesses a client with acquired immunodeficiency syndrome (AIDS) for signs and symptoms of cancer. What is the most common AIDS-related cancer? A. Multiple myeloma B. Squamous cell carcinoma C. Kaposi's sarcoma D. Leukemia

C. The CD4 count is the major indicator of immune function. Antiretroviral therapy in HIV targets different stages of the HIV life cycle. Therapy does not prevent opportunistic infections. Medication therapy is effective, and most clients respond well to it.

The nurse is reviewing the diagnostic test findings of a client with rheumatoid arthritis. What would the nurse expect to find? A. Increased C4 complement B. Increased red blood cell count C. Elevated erythrocyte sedimentation rate D. Increased albumin levels

C. The erythrocyte sedimentation rate (ESR) may be elevated, particularly as the disease progresses. ESR shows inflammation associated with RA. Red blood cell count and C4 complement component are decreased. Serum protein electrophoresis may disclose increased levels of gamma and alpha globulin but decreased albumin.

A client is newly diagnosed with rheumatoid arthritis. For which medications will the nurse prepare teaching for this client? Select all that apply. A) Prednisone B) Aspirin C) Ibuprofen D) Acetaminophen E) Methotrexate

A, C, E. Once the diagnosis of RA is made, treatment should begin with either a nonbiologic or biologic disease-modifying antirheumatic drugs (DMARD). The goal of using DMARD therapy is preventing inflammation and joint damage. Recommended treatment guidelines include beginning with the nonbiologic DMARD methotrexate as the preferred agent. Ibuprofen may be prescribed as an analgesic however must be used with caution because of the risk of gastric ulcer. Corticosteroids are recommended as a 'bridge' in the early treatment but are not recommended for long term therapy due to side effects. Aspirin and acetaminophen are not identified as medications used to treat the symptoms of RA.

A client is experiencing painful joints and changes in the lungs, heart, and kidneys. For which condition will the nurse schedule this client for diagnostic tests? A. Metabolic disorders B. Autoimmune disorders C. Vascular diseases D. Heart disease

B. A hallmark of inflammatory rheumatic diseases is autoimmunity, where the body mistakenly recognizes its own tissue as a foreign antigen. Although focused in the joints, inflammation and autoimmunity also involve other areas. The blood vessels (vasculitis and arteritis), lungs, heart, and kidneys may be affected by the autoimmunity and inflammation. It is unlikely that the client's array of symptoms is being caused by heart disease, vascular diseases, or metabolic disorders

A nurse assesses a client in the health care provider's office. Which assessment findings support a suspicion of systemic lupus erythematosus (SLE)? A. Weight gain, hypervigilance, hypothermia, and edema of the legs B. Facial erythema, pericarditis, pleuritis, fever, and weight loss C. Hypothermia, weight gain, lethargy, and edema of the arms D. Photosensitivity, polyarthralgia, and painful mucous membrane ulcers

B. An autoimmune disorder characterized by chronic inflammation of the connective tissues, SLE causes fever, weight loss, malaise, fatigue, skin rashes, and polyarthralgia. Nearly half of clients with SLE have facial erythema, (the classic butterfly rash). SLE also may cause profuse proteinuria (excretion of more than 0.5 g/day of protein), pleuritis, pericarditis, photosensitivity, and painless mucous membrane ulcers. Weight gain, hypervigilance, hypothermia, and edema of the legs and arms don't suggest SLE.

The registered nurse (RN) identifies what as the primary action of the drug epinephrine? A) Vasodilation B) Vasoconstriction C) Vaso-stability D) Vasorelaxation

B. Epinephrine is a sympathomimetic drug. This drug stimulates alpha and beta receptors, resulting in vasoconstriction. Epinephrine has a positive chronotropic and inotropic action resulting in increased cardiac force with contractions and increased heart rate. The two primary indicators for this drug include cardiac arrest and anaphylaxis. For cardiac arrest, the drug is administered as an intravenous push. For anaphylaxis, the drug is administered by an epinephrine auto-injector, also known as an 'epi-pen'. This drug is administered intramuscularly and has a rapid onset, but a 10 to 20 minute duration. Epinephrine is the priority drug of treatment for an anaphylactic event at the first sign of a severe allergic reaction (hives, dyspnea, hypotension).

A client presents at the clinic with an allergic disorder. The client asks the nurse what an allergic disorder means. What would be the nurse's best response? A. It is a muted response to something in the environment. B. It is a hyperimmune response to something in the environment that is usually harmless. C. It is a harmless reaction to something in the environment. D. It means you are very sensitive to something inside of yourself.

B. An allergic disorder is characterized by a hyperimmune response to weak antigens that usually are harmless. The antigens that can cause an allergic response are called allergens.


Set pelajaran terkait

Penny CH 24- Fetal Head and Brain

View Set

Personality Ch. 6 Learned Internal Drives Notes From Text

View Set

Abnormal Psychology Chapter 3, Abnormal Psychology Chapter 3 - quiz, Abnormal Psychology Chapter 3 - Assignment

View Set

UNIT 2 CIVIL WAR GROWTH OF AMERICA

View Set

Worksheet 24-2 - Digital Image Display

View Set

Cancer & Oncology Nursing NCLEX Practice Quiz

View Set

"Study Break Questions" from Ch 12, 20, 21 in Critical Care Book (Citation in Description)

View Set

PrepU Ch. 25: Structure and Function of the Cardiovascular System

View Set

Intro to Marketing Chapter 5 Consumer Markets and Consumer Buyer Behavior

View Set

Chapter 18 Policymaking and Domestic Policy

View Set